21
$\begingroup$

From Wiener's tauberian theorem we know that linear combinations of translates of f \in L^1(R) are dense in L^1(R) if and only if the Fourier transform of f never vanishes. It is also known that linear combinations of translates of f \in L^2(R) are dense in L^2 if and only if the Fourier transform of f is nonzero almost everywhere. Is there a characterization (in terms of the Fourier transform) of functions in L^p(R) with the property that linear combinations of its translates are dense in L^p?

If the answer is no can it be shown that no reasonable measure of the size of the zero set of the Fourier transform of f will suffice to give such a characterization?

$\endgroup$
5
  • 1
    $\begingroup$ This feels like it should be known (in the sense either of having a characterization, or a result showing no reasonable characterization is possible). Unfortunately I can't remember anything definite. Have you had any luck on MathSciNet? $\endgroup$
    – Yemon Choi
    Nov 5, 2009 at 6:35
  • $\begingroup$ Not really. This problem came up in something I was thinking about about a long while ago. At the time I did some fairly intensive detective work trying to find a reference/answer. I remember finding passing references to the problem but I never was able to track down an answer. $\endgroup$
    – Mark Lewko
    Nov 5, 2009 at 7:03
  • $\begingroup$ In $$ $$ math.tamu.edu/~thomas.schlumprecht/ossz.pdf $$ $$ Odell, Sari, Schlumprecht, and Zheng prove an interesting non Tauberian theorem: the translates of an $L_1$ function cannot contain a Schauder basis for $L_1$, nor can translates of an $L_p$ function contain an unconditional Schauder basis for $L_p$ when $p \le 4$. $\endgroup$ Mar 24, 2012 at 14:42
  • $\begingroup$ Is the reference that I give to this answer relevant to this question: mathoverflow.net/questions/333317/… ? $\endgroup$ Jun 6, 2019 at 11:03
  • $\begingroup$ ok, I saw that it is already referenced bellow, never mind. $\endgroup$ Jun 6, 2019 at 11:08

2 Answers 2

17
$\begingroup$

Actually this is a well known question. N. Lev and A. Olevskii have shown the following theorem:

Theorem (Lev, Olevskii) Given any 1 < p < 2 one can find two vectors in $l^1(Z)$, such that one is cyclic in $l^p(Z)$ and the other is not, but their Fourier transforms have an identical set of zeros.

The same result follows for $L^p(R)$.

Look here for example or on arxiv under Olevskii or Lev. This means more or less that for $p\neq 1,2$, there can be no characterization of $L^p$ generators in terms of the zero set of the Fourier transform. Hope this helps.

PS: Maybe I should add that I have the impression that this is a big open problem so you shouldn't expect an 'easy' answer. It is not clear in what terms one should seek for such a characterization. I would contact Nir Lev for more information (you can look for his e-mail on his web site).

$\endgroup$
3
  • $\begingroup$ Thanks! This explains why I thought the question "looked familiar" - I must have seen the abstract of something like this: arXiv:0908.0447 $\endgroup$
    – Yemon Choi
    Nov 5, 2009 at 19:25
  • 3
    $\begingroup$ Thanks! Looking at the paper of N. Lev and A. Olevskii I learned that there is a partial generalization of Beurling (On a closure problem, Ark. Mat. 1 (1951), pp. 301–303.) This states that the translates of f \in L^{p} is dense in L^p (for 1<p<2) if the Hausdorff dimension of the zero set of the Fourier transform of f is less than 2(p-1)/p. The converse, as has be pointed out, is false. $\endgroup$
    – Mark Lewko
    Nov 5, 2009 at 20:40
  • $\begingroup$ The link to sciencedirect.com is broken. I'm also unable to find any snapshot saved on the Wayback Machine. $\endgroup$ May 18, 2023 at 7:09
2
$\begingroup$

OK, my first naive thoughts are as follows. What follows is incomplete, but I'm leaving it here in case it suggests a proper solution or jogs someone's memory.

Let's look at the case 1< p < 2, and let f\in Lp(R) be such that translates of f do not span a dense subspace. By duality, there exists a nonzero g \in Lq(R) such that the convolution of f with g is zero (this convolution is a continuous function on R, so we don't need an a.e. qualifier here.)

Now if f and g were known to have well-defined Fourier transforms, the hypothesis that g is not identically zero ought to imply that the FT of f has to vanish on a `visible' subset of R. This suggests trying to introduce some mollifier functions, i.e. some h and k whose Fourier transforms are compactly supported but are 1 on very large intervals (maybe de la Vallee Poussin kernels would be enough?) and then considering

h*f*g*k = 0

where we hope that h*f and g*k will be in L1(R), and that g*k will not be identically zero. Then applying the previous argument we know that the FT of h*f would have to vanish on some open interval, and then by varying h maybe we can eventually say something about f...

$\endgroup$
1
  • $\begingroup$ If f is in $L^p$, $p\neq 1$, is there any mollifier $h$ which can make $h\ast f$ to be $L^1$? $\endgroup$
    – spr
    Mar 24, 2012 at 10:25

Your Answer

By clicking “Post Your Answer”, you agree to our terms of service and acknowledge you have read our privacy policy.

Not the answer you're looking for? Browse other questions tagged or ask your own question.